Bạn chưa đăng nhập. Vui lòng đăng nhập để hỏi bài

Những câu hỏi liên quan
bach nhac lam
Xem chi tiết
tthnew
25 tháng 4 2020 lúc 18:22

Câu c quen thuộc, chém trước:

Ta có BĐT phụ: \(\frac{x^3}{x^3+\left(y+z\right)^3}\ge\frac{x^4}{\left(x^2+y^2+z^2\right)^2}\) \((\ast)\)

Hay là: \(\frac{1}{x^3+\left(y+z\right)^3}\ge\frac{x}{\left(x^2+y^2+z^2\right)^2}\)

Có: \(8(y^2+z^2) \Big[(x^2 +y^2 +z^2)^2 -x\left\{x^3 +(y+z)^3 \right\}\Big]\)

\(= \left( 4\,x{y}^{2}+4\,x{z}^{2}-{y}^{3}-3\,{y}^{2}z-3\,y{z}^{2}-{z}^{3 } \right) ^{2}+ \left( 7\,{y}^{4}+8\,{y}^{3}z+18\,{y}^{2}{z}^{2}+8\,{z }^{3}y+7\,{z}^{4} \right) \left( y-z \right) ^{2} \)

Từ đó BĐT \((\ast)\) là đúng. Do đó: \(\sqrt{\frac{x^3}{x^3+\left(y+z\right)^3}}\ge\frac{x^2}{x^2+y^2+z^2}\)

\(\therefore VT=\sum\sqrt{\frac{x^3}{x^3+\left(y+z\right)^3}}\ge\sum\frac{x^2}{x^2+y^2+z^2}=1\)

Done.

zZz Cool Kid zZz
26 tháng 4 2020 lúc 11:26

Câu 1 chuyên phan bội châu

câu c hà nội

câu g khoa học tự nhiên

câu b am-gm dựa vào hằng đẳng thử rồi đặt ẩn phụ

câu f đặt \(a=\frac{2m}{n+p};b=\frac{2n}{p+m};c=\frac{2p}{m+n}\)

Gà như mình mấy câu còn lại ko bt nha ! để bạn tth_pro full cho nhé !

bach nhac lam
2 tháng 3 2020 lúc 23:47

Nguyễn Ngọc Lộc , ?Amanda?, Phạm Lan Hương, Akai Haruma, @Trần Thanh Phương, @Nguyễn Việt Lâm,

@tth_new

Giúp em vs ạ! Thanks nhiều ạ

Khách vãng lai đã xóa
nguyễn minh
Xem chi tiết
đào danh phước
7 tháng 1 2020 lúc 16:38

cho {a,b,c>0a+b+c=abc{a,b,c>0a+b+c=abc\left\{{}\begin{matrix}a,b,c>0\\a+b+c=abc\end{matrix}\right..CMR: ba2+cb2+ac2+3≥(1a+1b+1c)2+√3ba2+cb2+ac2+3≥(1a+1b+1c)2+3\frac{b}{a^2}+\frac{c}{b^2}+\frac{a}{c^2}+3\ge\left(\frac{1}{a}+\frac{1}{b}+\frac{1}{c}\right)^2+\sqrt{3}

Khách vãng lai đã xóa
đào danh phước
19 tháng 1 2020 lúc 19:10

cho {a,b,c>0a+b+c=abc{a,b,c>0a+b+c=abc\left\{{}\begin{matrix}a,b,c>0\\a+b+c=abc\end{matrix}\right..CMR: ba2+cb2+ac2+3≥(1a+1b+1c)2+√3ba2+cb2+ac2+3≥(1a+1b+1c)2+3\frac{b}{a^2}+\frac{c}{b^2}+\frac{a}{c^2}+3\ge\left(\frac{1}{a}+\frac{1}{b}+\frac{1}{c}\right)^2+\sqrt{3}

Khách vãng lai đã xóa
nguyễn minh
21 tháng 1 2020 lúc 18:01

Akai Haruma

Khách vãng lai đã xóa
bach nhac lam
Xem chi tiết
Nguyễn Việt Lâm
11 tháng 2 2020 lúc 22:21

Mới nghĩ ra 3 câu:

a/ \(\frac{ab}{\sqrt{\left(1-c\right)^2\left(1+c\right)}}=\frac{ab}{\sqrt{\left(a+b\right)^2\left(1+c\right)}}\le\frac{ab}{2\sqrt{ab\left(1+c\right)}}=\frac{1}{2}\sqrt{\frac{ab}{1+c}}\)

\(\sum\sqrt{\frac{ab}{1+c}}\le\sqrt{2\sum\frac{ab}{1+c}}\)

\(\sum\frac{ab}{1+c}=\sum\frac{ab}{a+c+b+c}\le\frac{1}{4}\sum\left(\frac{ab}{a+c}+\frac{ab}{b+c}\right)=\frac{1}{4}\)

c/ \(ab+bc+ca=2abc\Rightarrow\frac{1}{a}+\frac{1}{b}+\frac{1}{c}=2\)

Đặt \(\left(x;y;z\right)=\left(\frac{1}{a};\frac{1}{b};\frac{1}{c}\right)\Rightarrow x+y+z=2\)

\(VT=\sum\frac{x^3}{\left(2-x\right)^2}\)

Ta có đánh giá: \(\frac{x^3}{\left(2-x\right)^2}\ge x-\frac{1}{2}\) \(\forall x\in\left(0;2\right)\)

\(\Leftrightarrow2x^3\ge\left(2x-1\right)\left(x^2-4x+4\right)\)

\(\Leftrightarrow9x^2-12x+4\ge0\Leftrightarrow\left(3x-2\right)^2\ge0\)

d/ Ta có đánh giá: \(\frac{x^4+y^4}{x^3+y^3}\ge\frac{x+y}{2}\)

\(\Leftrightarrow\left(x-y\right)^2\left(x^2+xy+y^2\right)\ge0\)

Khách vãng lai đã xóa
bach nhac lam
11 tháng 2 2020 lúc 21:42

Akai Haruma, Nguyễn Ngọc Lộc , @tth_new, @Băng Băng 2k6, @Trần Thanh Phương, @Nguyễn Việt Lâm

Mn giúp e vs ạ! Thanks!

Khách vãng lai đã xóa
bach nhac lam
Xem chi tiết
Trần Thùy Linh
25 tháng 4 2020 lúc 13:04

\(A=\frac{a}{ab+c\left(a+b+c\right)}+\frac{b}{bc+a\left(a+b+c\right)}+\frac{c}{ca+b\left(a+b+c\right)}\)

\(=\frac{a}{\left(b+c\right)\left(a+c\right)}+\frac{b}{\left(a+b\right)\left(a+c\right)}+\frac{c}{\left(a+b\right)\left(c+b\right)}\)

Áp dụng bđt AM-GM ta có

\(A=\frac{a\left(a+b\right)+b\left(b+c\right)+c\left(c+a\right)}{\left(a+b\right)\left(b+c\right)\left(c+a\right)}\)

\(\ge27.\frac{a^2+b^2+c^2+ab+bc+ca}{8\left(a+b+c\right)^3}\)\(=\frac{a^2+b^2+c^2+ab+bc+ca}{8}\)

\(=\frac{\left(a+b+c\right)^2-\left(ab+bc+ca\right)}{8}\)\(\ge\frac{9-\frac{\left(a+b+c\right)^2}{3}}{8}=\frac{9-3}{8}=\frac{3}{4}\)

Dấu "=" xảy ra khi a=b=c=1

tthnew
25 tháng 4 2020 lúc 16:02

b) Mạnh hơn, và dễ dàng hơn là:

\(\left(\frac{a}{b}+\frac{b}{c}+\frac{c}{a}\right)^2\ge\left(a+b+c\right)\left(\frac{1}{a}+\frac{1}{b}+\frac{1}{c}\right)+\frac{\sum c\left(a-b\right)^2}{abc}\)

Nó tương đương với: \({\frac {{a}^{2}}{{b}^{2}}}+{\frac {{b}^{2}}{{c}^{2}}}+{\frac {{c}^{2} }{{a}^{2}}}+3-2\,{\frac {a}{b}}-2\,{\frac {b}{c}}-2\,{\frac {c}{a}} \geqq 0\)

Là hiển nhiên vì \(\frac{a^2}{b^2}+1\ge\frac{2a}{b}\)

Đơn giản:))

tthnew
25 tháng 4 2020 lúc 16:46

a) Đặt \(\left(x;y;z\right)=\left(\frac{1}{a};\frac{1}{b};\frac{1}{c}\right)\rightarrow ab+bc+ca=1;0< a,b,c< 1\)

Cần chứng minh: \(P=\sum\frac{\frac{1}{a}-1}{\frac{1}{b^2}}=\sum\frac{b^2-ab^2}{a}\ge\sqrt{3}-1\)

Hay là: \(\left(\frac{b^2}{a}+\frac{c^2}{b}+\frac{a^2}{c}\right)\sqrt{ab+bc+ca}\ge\left(\sqrt{3}-1\right)\left(ab+bc+ca\right)+a^2+b^2+c^2\)

\(\Leftrightarrow\left(\frac{b^2}{a}+\frac{c^2}{b}+\frac{a^2}{c}\right)^2\left(ab+bc+ca\right)\ge\) \(\Big[ (\sqrt{3} -1) (ab+bc+ca) +a^2+b^2+c^2\Big]^2\)

Giả sử \(c=\min\{a,b,c\}\) và đặt \(a=c+u, \, b=c+v \, (u,\, v \geq 0)\)

Nếu mình không nhìn nhầm, sau khi rút gọn, nhóm lại theo biến c, bạn nhận được một cái gì đó gọi là hiển nhiên haha

Chúc may mắn, mình mới rút gọn thử thì thấy có vẻ hiển nhiên thật :))

bach nhac lam
Xem chi tiết
Akai Haruma
1 tháng 12 2019 lúc 8:48

Bạn đã làm được rồi nhưng mình vẫn xin phép up lời giải nếu ai cần tham khảo:

Do $abc=1$ nên tồn tại $x,y,z>0$ sao cho $(a,b,c)=(\frac{x}{y}, \frac{y}{z}, \frac{z}{x})$

Khi đó, bài toán trở thành:

Cho $x,y,z>0$. CMR $A=\frac{y}{2y+x}+\frac{z}{2z+y}+\frac{x}{2x+z}\leq 1$

Thật vậy:

\(2A=\frac{2y}{2y+x}+\frac{2z}{2z+y}+\frac{2x}{2x+z}=1-\frac{x}{2y+x}+1-\frac{y}{2z+y}+1-\frac{z}{2x+z}\)

\(=3-\left(\frac{x}{x+2y}+\frac{y}{y+2z}+\frac{z}{z+2x}\right)\)

Áp dụng BĐT Cauchy-Schwarz:

\(\frac{x}{x+2y}+\frac{y}{y+2z}+\frac{z}{z+2x}=\frac{x^2}{x^2+2xy}+\frac{y^2}{y^2+2yz}+\frac{z^2}{z^2+2xz}\geq \frac{(x+y+z)^2}{x^2+2xy+y^2+2yz+z^2+2xz}=1\)

\(\Rightarrow 2A\leq 3-1=2\Rightarrow A\leq 1\) (đpcm)

Dấu "=" xảy ra khi $x=y=z$ hay $a=b=c=1$

Khách vãng lai đã xóa
bach nhac lam
30 tháng 11 2019 lúc 23:48

k cần nx ạ hehe

Khách vãng lai đã xóa
bach nhac lam
Xem chi tiết
bach nhac lam
10 tháng 11 2019 lúc 22:13

Vũ Minh Tuấn, HISINOMA KINIMADO, tth, Nguyễn Ngọc Linh, Hoàng Tử Hà, Aki Tsuki, @Akai Haruma,

@Nguyễn Việt Lâm

giúp e vs ạ!

thanks trước

Khách vãng lai đã xóa
Nguyễn Việt Lâm
10 tháng 11 2019 lúc 22:28

\(\Leftrightarrow\frac{2}{2+a}+\frac{2}{2+b}+\frac{2}{2+c}\le2\)

\(\Leftrightarrow\frac{a}{2+a}+\frac{b}{2+b}+\frac{c}{2+c}\ge1\)

Đặt \(\left(a;b;c\right)=\left(\frac{x}{y};\frac{y}{z};\frac{z}{x}\right)\)

\(\Rightarrow P=\frac{x}{x+2y}+\frac{y}{y+2z}+\frac{z}{z+2x}=\frac{x^2}{x^2+2xy}+\frac{y^2}{y^2+2yz}+\frac{z^2}{z^2+2zx}\)

\(\Rightarrow P\ge\frac{\left(x+y+z\right)^2}{x^2+y^2+z^2+2xy+2yz+2zx}=\frac{\left(x+y+z\right)^2}{\left(x+y+z\right)^2}=1\)

Dấu "=" xảy ra khi \(x=y=z\) hay \(a=b=c=1\)

Khách vãng lai đã xóa
tthnew
11 tháng 11 2019 lúc 7:31

Bài này em giải nhiều lần lắm rồi nên thôi đưa link nha..

Câu hỏi của khiêm nguyễn xuân - Toán lớp 9 - Học toán với OnlineMath (câu trả lời của tth_new:v)

Khách vãng lai đã xóa
bach nhac lam
Xem chi tiết
tthnew
8 tháng 8 2019 lúc 10:48

2a) Có cách này nhưng ko chắc!

\(A\ge\frac{4x^2}{y^2+z^2}+\frac{y^2+z^2}{x^2}=\frac{3x^2}{y^2+z^2}+\left(\frac{x^2}{y^2+z^2}+\frac{y^2+z^2}{x^2}\right)\)

\(\ge\frac{3\left(y^2+z^2\right)}{y^2+z^2}+2\sqrt{\frac{x^2}{y^2+z^2}.\frac{y^2+z^2}{x^2}}=3+2=5\)

Đẳng thức xảy ra khi x2 = y2 + z2????

bach nhac lam
8 tháng 8 2019 lúc 10:18

tth, ?Amanda?, @Nk>↑@, buithianhtho, Phạm Hoàng Lê Nguyên,

Akai Haruma, Aki Tsuki, @Nguyễn Việt Lâm, @Trần Thanh Phương

Giúp mk vs!

Akai Haruma
8 tháng 8 2019 lúc 11:48

1.

a) Ta thấy:

\(a^3+b^3-ab(a+b)=(a-b)^2(a+b)\geq 0, \forall a,b>0\)

\(\Rightarrow a^3+b^3\geq ab(a+b)\)

\(\Rightarrow \frac{19b^3-a^3}{ab+5b^2}=\frac{20b^3-(a^3+b^3)}{ab+5b^2}\leq \frac{20b^3-ab(a+b)}{ab+5b^2}=\frac{20b^2-a(a+b)}{a+5b}=\frac{(4b-a)(a+5b)}{a+5b}=4b-a\)

Hoàn toàn tương tự:

\(\frac{19c^3-b^3}{bc+5c^2}\leq 4c-b\); \(\frac{19a^3-c^3}{ac+5a^2}\leq 4a-c\)

Cộng theo vế và rút gọn:

\(\Rightarrow \sum \frac{19b^3-a^3}{ab+5b^2}\leq 3(a+b+c)\) (đpcm)

Dấu "=" xảy ra khi $a=b=c$

b) BĐT sai với $a=3,b=4$

Nguyễn Thu Ngà
Xem chi tiết
Nguyễn Thu Ngà
Xem chi tiết
bach nhac lam
Xem chi tiết
tthnew
11 tháng 12 2019 lúc 7:36

3 g) \(xyz=x+y+z+2\)

\(\Leftrightarrow\left(x+1\right)\left(y+1\right)\left(z+1\right)=\Sigma_{cyc}\left(x+1\right)\left(y+1\right)\)

\(\Rightarrow\frac{1}{x+1}+\frac{1}{y+1}+\frac{1}{z+1}=1\) .Đặt \(\frac{1}{x+1}=a;\frac{1}{y+1}=b;\frac{1}{z+1}=c\Rightarrow x=\frac{1-a}{a}=\frac{b+c}{a};y=\frac{c+a}{b};z=\frac{a+b}{c}\) vì a + b + c = 1.

Khi đó \(P=\Sigma_{cyc}\frac{1}{\sqrt{\frac{\left(b+c\right)^2}{a^2}+2}}=\Sigma_{cyc}\frac{a}{\sqrt{2a^2+\left(b+c\right)^2}}\)

\(=\sqrt{\frac{2}{9}+\frac{4}{9}}.\Sigma_{cyc}\frac{a}{\sqrt{\left[\left(\sqrt{\frac{2}{9}}\right)^2+\left(\sqrt{\frac{4}{9}}\right)^2\right]\left[2a^2+\left(b+c\right)^2\right]}}\)

\(\le\sqrt{\frac{2}{3}}\Sigma_{cyc}\frac{a}{\sqrt{\left[\frac{2}{3}a+\frac{2}{3}b+\frac{2}{3}c\right]^2}}=\frac{\sqrt{6}}{2}\left(a+b+c\right)=\frac{\sqrt{6}}{2}\)

Đẳng thức xảy ra khi \(a=b=c=\frac{1}{3}\Leftrightarrow x=y=z=2\)

Khách vãng lai đã xóa
tthnew
11 tháng 12 2019 lúc 7:43

3c) Nhìn quen quen, chả biết có lời giải ở đâu hay chưa nhưng vẫn làm:D (Em ko quan tâm nha!)

\(P=3-\Sigma_{cyc}\frac{2xy^2}{xy^2+xy^2+1}\ge3-\Sigma_{cyc}\frac{2xy^2}{3\sqrt[3]{\left(xy^2\right)^2}}=3-\frac{2}{3}\Sigma_{cyc}\sqrt[3]{\left(xy^2\right)}\)

\(\ge3-\frac{2}{3}\Sigma_{cyc}\frac{x+y+y}{3}=3-\frac{2}{3}\left(x+y+z\right)=3-2=1\)

Đẳng thức xảy ra khi \(x=y=z=\frac{1}{3}\)

Khách vãng lai đã xóa
bach nhac lam
10 tháng 12 2019 lúc 15:37

Vũ Minh Tuấn, Băng Băng 2k6, Nguyễn Lê Phước Thịnh, Phạm Lan Hương, Hưng Nguyễn Lê Việt, Nguyễn Việt Lâm, Nguyễn Thị Ngọc Thơ, Phạm Băng Nguyệt, HISINOMA KINIMADO, @Akai Haruma, @Trần Thanh Phương, @tth_new

giúp e vs ạ! thanks nhiều!

Khách vãng lai đã xóa